Diễn Đàn MathScopeDiễn Đàn MathScope
  Diễn Đàn MathScope
Ghi Danh Hỏi/Ðáp Community Lịch

Go Back   Diễn Đàn MathScope > Sơ Cấp > Việt Nam và IMO > 2015

News & Announcements

Ngoài một số quy định đã được nêu trong phần Quy định của Ghi Danh , mọi người tranh thủ bỏ ra 5 phút để đọc thêm một số Quy định sau để khỏi bị treo nick ở MathScope nhé !

* Nội quy MathScope.Org

* Một số quy định chung !

* Quy định về việc viết bài trong diễn đàn MathScope

* Nếu bạn muốn gia nhập đội ngũ BQT thì vui lòng tham gia tại đây

* Những câu hỏi thường gặp

* Về việc viết bài trong Box Đại học và Sau đại học


Trả lời Gởi Ðề Tài Mới
 
Ðiều Chỉnh Xếp Bài
Old 08-01-2015, 02:35 PM   #1
magic.
+Thành Viên+
 
Tham gia ngày: Aug 2010
Bài gởi: 213
Thanks: 107
Thanked 140 Times in 84 Posts
Trích:
Nguyên văn bởi quocbaoct10 View Post
Nhìn câu tổ hợp mà mình muốn khóc, tại sao đề thi HSG mà lại ra cái câu như thế này ?
Trích:
Nguyên văn bởi quocbaoct10 View Post
Được bạn. Hình như có thể chia thành 2 dãy, $a_n$ chia hết cho 3 và $b_n$ không chia hết cho 3. Công thức truy hồi cũng không khó tính.
Từ cách gọi dãy như mình ở trên, ta được 2 công thức:
$a_{n+1}=a_n+3b_n$ và $a_n+b_n=4^n$.
Đến đây thì dễ rồi.
Bạn xem lại công thức truy hồi nhé!
[RIGHT][I][B]Nguồn: MathScope.ORG[/B][/I][/RIGHT]
 
__________________
Peace195
magic. is offline   Trả Lời Với Trích Dẫn
Old 08-01-2015, 02:47 PM   #2
quocbaoct10
+Thành Viên Danh Dự+
 
quocbaoct10's Avatar
 
Tham gia ngày: Oct 2012
Đến từ: THPT chuyên Lê Quý Đôn-Nha Trang-Khánh Hòa
Bài gởi: 539
Thanks: 292
Thanked 365 Times in 217 Posts
Trích:
Nguyên văn bởi magic. View Post
Bạn xem lại công thức truy hồi nhé!
Đúng là nhầm thật, cảm ơn anh.
[RIGHT][I][B]Nguồn: MathScope.ORG[/B][/I][/RIGHT]
 
__________________
i'll try my best.
quocbaoct10 is offline   Trả Lời Với Trích Dẫn
Old 08-01-2015, 12:12 PM   #3
lupanh7
+Thành Viên+
 
Tham gia ngày: Jul 2011
Bài gởi: 10
Thanks: 7
Thanked 6 Times in 3 Posts
Bài 3 làm truy hồi cũng không khó lắm
Kết quả là:
$\frac{4^K-1}{3}+1$ nếu K chia hết cho 3
$\frac{4^K-1}{3}$ nếu K không chia hết cho 3
[RIGHT][I][B]Nguồn: MathScope.ORG[/B][/I][/RIGHT]
 

thay đổi nội dung bởi: lupanh7, 08-01-2015 lúc 12:45 PM
lupanh7 is offline   Trả Lời Với Trích Dẫn
The Following User Says Thank You to lupanh7 For This Useful Post:
thaygiaocht (08-01-2015)
Old 08-01-2015, 12:22 PM   #4
Nguyen Van Linh
Moderator
 
Tham gia ngày: Aug 2009
Đến từ: Hà Nội
Bài gởi: 277
Thanks: 69
Thanked 323 Times in 145 Posts
Câu b bài hình:
Xét trục đẳng phương cho $(I)$, đường tròn đường kính $BC$ và $(BHC)$ suy ra $EF, PQ, BC$ đông quy tại $K$. Kẻ tiếp tuyến $KT$ tới $(O)$.
Ta có $KM.KN=KP.KQ=KB.KC=KT^2$ nên $(TPQ)$ tiếp xúc với $(O)$, $(TMN)$ tiếp xúc với $(O).$
Ta có $\angle BTM=\angle KTM-\angle KTB=\angle TNM-\angle TCN=\angle NTC$. Suy ra $TM,TN$ đẳng giác trong $\angle BTC$ hay phân giác $MTN$ đi qua trung điểm cung $BC$ không chứa $A.$
[RIGHT][I][B]Nguồn: MathScope.ORG[/B][/I][/RIGHT]
 
Hình Kèm Theo
Kiểu File : png Hinh.png (35.6 KB, 95 lần tải)

thay đổi nội dung bởi: Nguyen Van Linh, 08-01-2015 lúc 12:29 PM
Nguyen Van Linh is offline   Trả Lời Với Trích Dẫn
The Following 5 Users Say Thank You to Nguyen Van Linh For This Useful Post:
dangvip123tb (09-01-2015), DenisO (08-01-2015), huynhcongbang (08-01-2015), nhatduyt1k24 (09-01-2015), Unknowing (08-01-2015)
Old 08-01-2015, 12:26 PM   #5
huynhcongbang
Administrator

 
huynhcongbang's Avatar
 
Tham gia ngày: Feb 2009
Đến từ: Ho Chi Minh City
Bài gởi: 2,413
Thanks: 2,165
Thanked 4,188 Times in 1,381 Posts
Gửi tin nhắn qua Yahoo chát tới huynhcongbang
Gửi mọi người đề thi ngày 1, bản PDF đọc cho rõ.
[RIGHT][I][B]Nguồn: MathScope.ORG[/B][/I][/RIGHT]
 
File Kèm Theo
Kiểu File : pdf VMO 2015 Ngay 1.pdf (313.1 KB, 789 lần tải)
__________________
Sự im lặng của bầy mèo
huynhcongbang is offline   Trả Lời Với Trích Dẫn
The Following 5 Users Say Thank You to huynhcongbang For This Useful Post:
9nho10mong (08-01-2015), n.t.tuan (09-01-2015), thaygiaocht (09-01-2015), Toan95cqb (08-01-2015), zinxinh (10-01-2015)
Old 08-01-2015, 12:47 PM   #6
huynhcongbang
Administrator

 
huynhcongbang's Avatar
 
Tham gia ngày: Feb 2009
Đến từ: Ho Chi Minh City
Bài gởi: 2,413
Thanks: 2,165
Thanked 4,188 Times in 1,381 Posts
Gửi tin nhắn qua Yahoo chát tới huynhcongbang
Câu 3 tổ hợp theo mình nghĩ thì nếu ai học về đếm bằng truy hồi thì cũng đã từng gặp bài tương tự rồi.

Ở đây tuy đề có yêu cầu số chữ số không vượt quá $K$ nhưng may mắn là trong các số cần lập thì có số 0. Điều này cho phép ta đưa về bài toán:

Đếm số các bộ số có thứ tự có đúng $K$ số, có tổng chia hết cho 3 và có các chữ số là $\{ 2, 0, 1, 5 \}$ (*)

Chú ý rằng cách đếm trên có thể có các chữ số 0 đứng đầu và thông qua đó, ta đã gián tiếp đếm các số có ít hơn $K$ chữ số.

Cách của bạn haojack123 vậy là ổn rồi, nhưng mình nghĩ là nên rút gọn ra đến kết quả cuối cùng luôn, do đó đều là các tổng của các cấp số nhân đơn giản.

Câu 2, ý 2 dành cho bạn nào quên BĐT Schur bậc 4.

Đặt $\sqrt{a}=x,\sqrt{b}=y,\sqrt{c}=z$ thì BĐT thứ hai chính là:
\[\begin{align}
& ({{x}^{2}}+{{y}^{2}}+{{z}^{2}})(xy+yz+zx)+{{({{x}^ {2}}-{{y}^{2}})}^{2}}+{{({{y}^{2}}-{{z}^{2}})}^{2}}+{{({{z}^{2}}-{{x}^{2}})}^{2}}\ge {{({{x}^{2}}+{{y}^{2}}+{{z}^{2}})}^{2}} \\
& \Leftrightarrow {{({{x}^{2}}-{{y}^{2}})}^{2}}+{{({{y}^{2}}-{{z}^{2}})}^{2}}+{{({{z}^{2}}-{{x}^{2}})}^{2}}\ge ({{x}^{2}}+{{y}^{2}}+{{z}^{2}})\left( {{x}^{2}}+{{y}^{2}}+{{z}^{2}}-xy-yz-zx \right) \\
& \Leftrightarrow {{(x-y)}^{2}}{{(x+y)}^{2}}+{{(y-z)}^{2}}{{(y+z)}^{2}}+{{(z-x)}^{2}}{{(z+x)}^{2}} \\
& \ge \frac{1}{2}({{x}^{2}}+{{y}^{2}}+{{z}^{2}})\left( {{(x-y)}^{2}}+{{(y-z)}^{2}}+{{(z-x)}^{2}} \right) \\
& \Leftrightarrow \sum{{{(x-y)}^{2}}\left( 2{{(x+y)}^{2}}-({{x}^{2}}+{{y}^{2}}+{{z}^{2}}) \right)}\ge 0 \\
& \Leftrightarrow \sum{{{(x-y)}^{2}}\left( {{x}^{2}}+{{y}^{2}}+4xy-{{z}^{2}} \right)}\ge 0 \\
\end{align}\]
Giả sử $x\ge y\ge z$ thì rõ ràng $\left\{ \begin{align}
& {{x}^{2}}+{{y}^{2}}+4xy-{{z}^{2}}\ge 0 \\
& {{z}^{2}}+{{x}^{2}}+4zx-{{y}^{2}}\ge 0 \\
\end{align} \right.$ . Ta có \[\left| y-z \right|\le \left| x-z \right|\Rightarrow {{(y-z)}^{2}}\le {{(x-z)}^{2}}\] nên ta có
\[{{(z-x)}^{2}}\left( {{z}^{2}}+{{x}^{2}}+4zx-{{y}^{2}} \right)+{{(y-z)}^{2}}\left( {{y}^{2}}+{{z}^{2}}+4yz-{{x}^{2}} \right)\ge {{(y-z)}^{2}}(2{{z}^{2}}+4zx+4yz)\] và $${{(x-y)}^{2}}({{x}^{2}}+{{y}^{2}}+4xy-{{z}^{2}})\ge 0.$$ Do đó, ta có đpcm.

Đẳng thức xảy ra khi $x=y=z$ hoặc $x=y, z=0$ và các hoán vị.
[RIGHT][I][B]Nguồn: MathScope.ORG[/B][/I][/RIGHT]
 
__________________
Sự im lặng của bầy mèo

thay đổi nội dung bởi: huynhcongbang, 08-01-2015 lúc 01:33 PM
huynhcongbang is offline   Trả Lời Với Trích Dẫn
The Following 4 Users Say Thank You to huynhcongbang For This Useful Post:
CTK9 (08-01-2015), DenisO (08-01-2015), n.v.thanh (10-01-2015), thaygiaocht (08-01-2015)
Old 08-01-2015, 12:30 PM   #7
Tarantallegra
+Thành Viên+
 
Tham gia ngày: Mar 2014
Bài gởi: 1
Thanks: 0
Thanked 0 Times in 0 Posts
Có ai giải ý a bài 1 bằng Lagrange không ạ?
[RIGHT][I][B]Nguồn: MathScope.ORG[/B][/I][/RIGHT]
 
Tarantallegra is offline   Trả Lời Với Trích Dẫn
Old 08-01-2015, 12:27 PM   #8
haojack123
+Thành Viên+
 
Tham gia ngày: Jun 2013
Bài gởi: 7
Thanks: 6
Thanked 23 Times in 6 Posts
Vì $n\le {{10}^{k}}$ nên suy ra $n=\overline{{{a}_{1}}{{a}_{2}}...{{a}_{k}}}$ với ${{a}_{i}}\in \left\{ 2,1,0,5 \right\}.$
Ta có $n\vdots 3\Leftrightarrow {{a}_{1}}+{{a}_{2}}+...+{{a}_{k}}\vdots 3.$
Gọi ${{A}_{k}}$ là số tập $\left( {{a}_{1}},{{a}_{2}},...,{{a}_{k}} \right)$ với ${{a}_{i}}\in \left\{ 2,1,0,5 \right\}$ mà $\sum\limits_{i=}^{k}{{{a}_{i}}\equiv 1\left( \bmod 3 \right).}$
Gọi ${{B}_{k}}$ là số tập $\left( {{a}_{1}},{{a}_{2}},...,{{a}_{k}} \right)$ với ${{a}_{i}}\in \left\{ 2,1,0,5 \right\}$ mà $\sum\limits_{i=}^{k}{{{a}_{i}}\equiv 2\left( \bmod 3 \right).}$
Gọi ${{C}_{k}}$ là số tập $\left( {{a}_{1}},{{a}_{2}},...,{{a}_{k}} \right)$ với ${{a}_{i}}\in \left\{ 2,1,0,5 \right\}$ mà $\sum\limits_{i=}^{k}{{{a}_{i}}\equiv 0\left( \bmod 3 \right).}$
Ứng với một tập con của ${{C}_{k}}$ thì ta được một số tự nhiên $n$ thoả mãn bài toán.
Khi đó số các số tự nhiên cần tìm là ${{C}_{k}}.$
Ngoài ra ta có: ${{A}_{1}}=1,{{A}_{2}}=4,{{B}_{1}}=2,{{B}_{2}}=4,{ {C}_{1}}=1,{{C}_{2}}=4.$
Lại có: \[\left\{ \begin{align}
& {{C}_{k}}=2{{A}_{k-1}}+{{B}_{k-1}}+{{C}_{k-1}} \\
& {{A}_{k}}={{A}_{k-1}}+2{{B}_{k-1}}+{{C}_{k-1}} \\
& {{B}_{k}}={{A}_{k-1}}+{{B}_{k-1}}+2{{C}_{k-1}} \\
\end{align} \right.\]
Suy ra: \[{{C}_{k}}={{C}_{k-3}}+3\left( {{4}^{k-2}}+{{4}^{k-3}}+{{4}^{k-4}} \right)\]
[RIGHT][I][B]Nguồn: MathScope.ORG[/B][/I][/RIGHT]
 
haojack123 is offline   Trả Lời Với Trích Dẫn
The Following 6 Users Say Thank You to haojack123 For This Useful Post:
CTK9 (08-01-2015), huynhcongbang (08-01-2015), nhatduyt1k24 (09-01-2015), nqt (08-01-2015), thaygiaocht (08-01-2015), thiendieu96 (09-01-2015)
Old 08-01-2015, 12:34 PM   #9
Nguyen Van Linh
Moderator
 
Tham gia ngày: Aug 2009
Đến từ: Hà Nội
Bài gởi: 277
Thanks: 69
Thanked 323 Times in 145 Posts
Bài hình có thể tổng quát:
Cho tam giác $ABC$ nội tiếp $(O).$ $(O_1)$ là đường tròn bất kì qua $B,C$. . Một đường tròn $\omega$ thay đổi cắt $BC$ tại $M,N$, cắt $(O_1)$ tại $P,Q$. Đường tròn đi qua $P,Q$ và tiếp xúc với $(O)$ tại $T$. Khi đó phân giác $\angle MTN$ luôn đi qua trung điểm cung $BC$ không chứa $A.$
[RIGHT][I][B]Nguồn: MathScope.ORG[/B][/I][/RIGHT]
 

thay đổi nội dung bởi: Nguyen Van Linh, 08-01-2015 lúc 12:40 PM
Nguyen Van Linh is offline   Trả Lời Với Trích Dẫn
The Following 3 Users Say Thank You to Nguyen Van Linh For This Useful Post:
dangvip123tb (10-01-2015), huynhcongbang (08-01-2015), thaygiaocht (08-01-2015)
Old 08-01-2015, 12:32 PM   #10
BlackSelena
+Thành Viên+
 
BlackSelena's Avatar
 
Tham gia ngày: Aug 2012
Bài gởi: 40
Thanks: 22
Thanked 18 Times in 14 Posts
Câu a bài hình chỉ cần chú ý với mọi $(I)$, nếu gọi giao điểm của $(I)$ với $BE, CF$ là $X,Y$ thì có $XY \parallel BC$.
Câu b mình làm không khác mấy anh LTL ở trên.
P/s: ai làm câu b bài dãy đi
[RIGHT][I][B]Nguồn: MathScope.ORG[/B][/I][/RIGHT]
 
BlackSelena is offline   Trả Lời Với Trích Dẫn
The Following User Says Thank You to BlackSelena For This Useful Post:
beppkid (08-01-2015)
Old 08-01-2015, 12:33 PM   #11
hoathuy21990
+Thành Viên+
 
Tham gia ngày: Apr 2014
Bài gởi: 18
Thanks: 4
Thanked 6 Times in 6 Posts
Bất đẳng thức có vẻ nhẹ nhàng hơn năm trước
[RIGHT][I][B]Nguồn: MathScope.ORG[/B][/I][/RIGHT]
 
hoathuy21990 is offline   Trả Lời Với Trích Dẫn
The Following User Says Thank You to hoathuy21990 For This Useful Post:
thaygiaocht (08-01-2015)
Old 08-01-2015, 12:51 PM   #12
quocbaoct10
+Thành Viên Danh Dự+
 
quocbaoct10's Avatar
 
Tham gia ngày: Oct 2012
Đến từ: THPT chuyên Lê Quý Đôn-Nha Trang-Khánh Hòa
Bài gởi: 539
Thanks: 292
Thanked 365 Times in 217 Posts
Câu dãy có vẻ có thể mở rộng ra với mọi giá trị $a$. Viết lại được thành:
$u_{n+1}=\frac{u_n}{2}+\frac{1}{4+\frac{a}{n^2}}. \sqrt{u_n^2+3}$.
vì $\lim_{n \to +\inf} \frac{a}{n^2}=0$ nên từ đó có thể suy ra rằng dãy số ở câu b đến một lúc nào đó sẽ thành dãy số như ở câu a, và dãy này thì luôn có giới hạn.
[RIGHT][I][B]Nguồn: MathScope.ORG[/B][/I][/RIGHT]
 
__________________
i'll try my best.
quocbaoct10 is offline   Trả Lời Với Trích Dẫn
The Following User Says Thank You to quocbaoct10 For This Useful Post:
thaygiaocht (08-01-2015)
Old 08-01-2015, 12:51 PM   #13
tuankietpq
+Thành Viên+
 
tuankietpq's Avatar
 
Tham gia ngày: Jul 2014
Đến từ: Trên mặt đất, dưới mặt trời
Bài gởi: 220
Thanks: 48
Thanked 118 Times in 80 Posts
Đi thi hsg quốc gia có được dùng hàm sinh không?
[RIGHT][I][B]Nguồn: MathScope.ORG[/B][/I][/RIGHT]
 
__________________
Kẻ mạnh đôi khi không phải là kẻ chiến thắng mà kẻ chiến thắng mới là kẻ mạnh.
tuankietpq is offline   Trả Lời Với Trích Dẫn
The Following User Says Thank You to tuankietpq For This Useful Post:
thaygiaocht (08-01-2015)
Old 08-01-2015, 02:04 PM   #14
quocbaoct10
+Thành Viên Danh Dự+
 
quocbaoct10's Avatar
 
Tham gia ngày: Oct 2012
Đến từ: THPT chuyên Lê Quý Đôn-Nha Trang-Khánh Hòa
Bài gởi: 539
Thanks: 292
Thanked 365 Times in 217 Posts
Trích:
Nguyên văn bởi tuankietpq View Post
Đi thi hsg quốc gia có được dùng hàm sinh không?
Không bạn nhé.
[RIGHT][I][B]Nguồn: MathScope.ORG[/B][/I][/RIGHT]
 
__________________
i'll try my best.
quocbaoct10 is offline   Trả Lời Với Trích Dẫn
The Following User Says Thank You to quocbaoct10 For This Useful Post:
tuankietpq (08-01-2015)
Old 08-01-2015, 03:39 PM   #15
analysis90
+Thành Viên+
 
Tham gia ngày: Jan 2011
Bài gởi: 89
Thanks: 46
Thanked 39 Times in 23 Posts
Trích:
Nguyên văn bởi quocbaoct10 View Post
Không bạn nhé.
Em có thể giải thích tại sao không? nó chỉ đơn thuần là bài toán về đa thức và số phức. Bài tổ hợp này thực chất là bài tổ hợp mà trường PTNK đã từng cho thi và cũng là bài tổ hợp của Romani năm 2003.
[RIGHT][I][B]Nguồn: MathScope.ORG[/B][/I][/RIGHT]
 
analysis90 is offline   Trả Lời Với Trích Dẫn
The Following 2 Users Say Thank You to analysis90 For This Useful Post:
buivanloc (08-01-2015), DenisO (08-01-2015)
Trả lời Gởi Ðề Tài Mới

Bookmarks


Quuyền Hạn Của Bạn
You may not post new threads
You may not post replies
You may not post attachments
You may not edit your posts

BB code is Mở
Smilies đang Mở
[IMG] đang Mở
HTML đang Tắt

Chuyển đến


Múi giờ GMT. Hiện tại là 02:29 AM.


Powered by: vBulletin Copyright ©2000-2024, Jelsoft Enterprises Ltd.
Inactive Reminders By mathscope.org
[page compression: 110.18 k/127.41 k (13.53%)]